For $a_n$ positive, $prod_n frac1{1+a_n}=0$ if and only if $sum_n a_n=infty$.












4












$begingroup$


This makes sense intuitively but I am not quite sure how to prove it rigorously. Any hints on how to proceed?










share|cite|improve this question









$endgroup$








  • 1




    $begingroup$
    It is helpful to note that, for $a_n$ sufficiently small, we have $frac 1{1+a_n} = 1 - a_n + a_n^2 + cdots approx 1 - a_n$
    $endgroup$
    – Omnomnomnom
    Dec 31 '18 at 20:40








  • 4




    $begingroup$
    Alternatively, I find it easier to look at the log $$ logleft[ prod_n frac 1{1 + a_n}right] = -sum_n log(1 + a_n) $$
    $endgroup$
    – Omnomnomnom
    Dec 31 '18 at 20:43










  • $begingroup$
    @Omnomnomnom Ah of course, I was forgetting that simplification with logarithm. Thanks!
    $endgroup$
    – Math1000
    Dec 31 '18 at 20:48










  • $begingroup$
    It's in the old classic Infinite Sequences And Series, by Bromwich, which I think is available cheap from Dover Publications. This book has no pre-requisite (e.g. no calculus needed). Some editions have slightly different titles. A "companion" theorem is: If $0leq a_n<1$ for all $n$ then $ prod_{n=1}^{infty}(1-a_n)=0 $ iff $ sum_{n=1}^{infty}a_n=infty.$
    $endgroup$
    – DanielWainfleet
    Dec 31 '18 at 20:57


















4












$begingroup$


This makes sense intuitively but I am not quite sure how to prove it rigorously. Any hints on how to proceed?










share|cite|improve this question









$endgroup$








  • 1




    $begingroup$
    It is helpful to note that, for $a_n$ sufficiently small, we have $frac 1{1+a_n} = 1 - a_n + a_n^2 + cdots approx 1 - a_n$
    $endgroup$
    – Omnomnomnom
    Dec 31 '18 at 20:40








  • 4




    $begingroup$
    Alternatively, I find it easier to look at the log $$ logleft[ prod_n frac 1{1 + a_n}right] = -sum_n log(1 + a_n) $$
    $endgroup$
    – Omnomnomnom
    Dec 31 '18 at 20:43










  • $begingroup$
    @Omnomnomnom Ah of course, I was forgetting that simplification with logarithm. Thanks!
    $endgroup$
    – Math1000
    Dec 31 '18 at 20:48










  • $begingroup$
    It's in the old classic Infinite Sequences And Series, by Bromwich, which I think is available cheap from Dover Publications. This book has no pre-requisite (e.g. no calculus needed). Some editions have slightly different titles. A "companion" theorem is: If $0leq a_n<1$ for all $n$ then $ prod_{n=1}^{infty}(1-a_n)=0 $ iff $ sum_{n=1}^{infty}a_n=infty.$
    $endgroup$
    – DanielWainfleet
    Dec 31 '18 at 20:57
















4












4








4


1



$begingroup$


This makes sense intuitively but I am not quite sure how to prove it rigorously. Any hints on how to proceed?










share|cite|improve this question









$endgroup$




This makes sense intuitively but I am not quite sure how to prove it rigorously. Any hints on how to proceed?







real-analysis sequences-and-series






share|cite|improve this question













share|cite|improve this question











share|cite|improve this question




share|cite|improve this question










asked Dec 31 '18 at 20:39









Math1000Math1000

19.4k31746




19.4k31746








  • 1




    $begingroup$
    It is helpful to note that, for $a_n$ sufficiently small, we have $frac 1{1+a_n} = 1 - a_n + a_n^2 + cdots approx 1 - a_n$
    $endgroup$
    – Omnomnomnom
    Dec 31 '18 at 20:40








  • 4




    $begingroup$
    Alternatively, I find it easier to look at the log $$ logleft[ prod_n frac 1{1 + a_n}right] = -sum_n log(1 + a_n) $$
    $endgroup$
    – Omnomnomnom
    Dec 31 '18 at 20:43










  • $begingroup$
    @Omnomnomnom Ah of course, I was forgetting that simplification with logarithm. Thanks!
    $endgroup$
    – Math1000
    Dec 31 '18 at 20:48










  • $begingroup$
    It's in the old classic Infinite Sequences And Series, by Bromwich, which I think is available cheap from Dover Publications. This book has no pre-requisite (e.g. no calculus needed). Some editions have slightly different titles. A "companion" theorem is: If $0leq a_n<1$ for all $n$ then $ prod_{n=1}^{infty}(1-a_n)=0 $ iff $ sum_{n=1}^{infty}a_n=infty.$
    $endgroup$
    – DanielWainfleet
    Dec 31 '18 at 20:57
















  • 1




    $begingroup$
    It is helpful to note that, for $a_n$ sufficiently small, we have $frac 1{1+a_n} = 1 - a_n + a_n^2 + cdots approx 1 - a_n$
    $endgroup$
    – Omnomnomnom
    Dec 31 '18 at 20:40








  • 4




    $begingroup$
    Alternatively, I find it easier to look at the log $$ logleft[ prod_n frac 1{1 + a_n}right] = -sum_n log(1 + a_n) $$
    $endgroup$
    – Omnomnomnom
    Dec 31 '18 at 20:43










  • $begingroup$
    @Omnomnomnom Ah of course, I was forgetting that simplification with logarithm. Thanks!
    $endgroup$
    – Math1000
    Dec 31 '18 at 20:48










  • $begingroup$
    It's in the old classic Infinite Sequences And Series, by Bromwich, which I think is available cheap from Dover Publications. This book has no pre-requisite (e.g. no calculus needed). Some editions have slightly different titles. A "companion" theorem is: If $0leq a_n<1$ for all $n$ then $ prod_{n=1}^{infty}(1-a_n)=0 $ iff $ sum_{n=1}^{infty}a_n=infty.$
    $endgroup$
    – DanielWainfleet
    Dec 31 '18 at 20:57










1




1




$begingroup$
It is helpful to note that, for $a_n$ sufficiently small, we have $frac 1{1+a_n} = 1 - a_n + a_n^2 + cdots approx 1 - a_n$
$endgroup$
– Omnomnomnom
Dec 31 '18 at 20:40






$begingroup$
It is helpful to note that, for $a_n$ sufficiently small, we have $frac 1{1+a_n} = 1 - a_n + a_n^2 + cdots approx 1 - a_n$
$endgroup$
– Omnomnomnom
Dec 31 '18 at 20:40






4




4




$begingroup$
Alternatively, I find it easier to look at the log $$ logleft[ prod_n frac 1{1 + a_n}right] = -sum_n log(1 + a_n) $$
$endgroup$
– Omnomnomnom
Dec 31 '18 at 20:43




$begingroup$
Alternatively, I find it easier to look at the log $$ logleft[ prod_n frac 1{1 + a_n}right] = -sum_n log(1 + a_n) $$
$endgroup$
– Omnomnomnom
Dec 31 '18 at 20:43












$begingroup$
@Omnomnomnom Ah of course, I was forgetting that simplification with logarithm. Thanks!
$endgroup$
– Math1000
Dec 31 '18 at 20:48




$begingroup$
@Omnomnomnom Ah of course, I was forgetting that simplification with logarithm. Thanks!
$endgroup$
– Math1000
Dec 31 '18 at 20:48












$begingroup$
It's in the old classic Infinite Sequences And Series, by Bromwich, which I think is available cheap from Dover Publications. This book has no pre-requisite (e.g. no calculus needed). Some editions have slightly different titles. A "companion" theorem is: If $0leq a_n<1$ for all $n$ then $ prod_{n=1}^{infty}(1-a_n)=0 $ iff $ sum_{n=1}^{infty}a_n=infty.$
$endgroup$
– DanielWainfleet
Dec 31 '18 at 20:57






$begingroup$
It's in the old classic Infinite Sequences And Series, by Bromwich, which I think is available cheap from Dover Publications. This book has no pre-requisite (e.g. no calculus needed). Some editions have slightly different titles. A "companion" theorem is: If $0leq a_n<1$ for all $n$ then $ prod_{n=1}^{infty}(1-a_n)=0 $ iff $ sum_{n=1}^{infty}a_n=infty.$
$endgroup$
– DanielWainfleet
Dec 31 '18 at 20:57












3 Answers
3






active

oldest

votes


















4












$begingroup$

I assume $lim_na_n=0$ otherwise it is trivial: then
$$
lnPi_nbig({1over{1+ a_n}}big)=sum -ln(1+a_n),
$$

and this is equivalent to $-sum a_n$ since
$$ln(1+a_n)simeq a_ntext{ if }lim_na_n=0.$$






share|cite|improve this answer











$endgroup$













  • $begingroup$
    So $-sum_nlog(1+a_n)=-infty$ would imply that the product is zero, and being finite would imply that the product is positive? Thanks.
    $endgroup$
    – Math1000
    Dec 31 '18 at 20:55



















1












$begingroup$

Let $a_ngeq 0$ for all $n.$ By induction on $n$ we have $$prod_{j=1}(1+a_j)geq 1+sum_{j=1}^na_j.$$ So if the sum diverges then so does the product.



We have $exp(a_j)geq 1+a_j, $ so $$exp (sum_{j=1}^na_j)geq prod_{j=1}^n(1+a_j).$$ So if the sum converges then so does the product.






share|cite|improve this answer









$endgroup$













  • $begingroup$
    I think you're looking at $prod_j (1+a_j)$ as opposed to $prod_j frac1{1+a_j}$ here.
    $endgroup$
    – Math1000
    Dec 31 '18 at 22:05



















0












$begingroup$

Fact 1: For any $xgeq0$, we have $ln(1+x)leq x$.



Proof: Let $f:[0,infty)rightarrowmathbb{R}$ be defined by $f(x)=x-ln(1+x)$.
Observe that $f'(x)=frac{x}{1+x}geq0,$ so $f$ is increasing. For
any $xin[0,infty)$, we have $f(x)geq f(0)=0$.



/////////////////////////////////



Fact 2: For any $xin[0,1]$, we have $ln(1+x)geqfrac{x}{2}$.



Proof: Let $g:[0,1]rightarrowmathbb{R}$ be defined by $g(x)=ln(1+x)-frac{x}{2}$.
Observe that $g'(x)=frac{1-x}{2(1+x)}geq0$, so $g$ is increasing.
For any $xin[0,1]$, we have $g(x)geq g(0)=0$.



/////////////////////////////////



Let $P_{n}=prod_{k=1}^{n}frac{1}{1+a_{k}}$. Then
begin{eqnarray*}
-ln P_{n} & = & sum_{k=1}^{n}ln(1+a_{k})\
& leq & sum_{k=1}^{n}a_{k}.
end{eqnarray*}

If $P_{n}rightarrow0$, we have $-ln P_{n}rightarrowinfty$, so
$sum_{k=1}^{infty}a_{k}=infty$.



/////////////////////////////////////////////////



For each $k$, let $b_{k}=min(a_{k},1)leq a_{k}$. Then,
begin{eqnarray*}
-ln P_{n} & = & sum_{k=1}^{n}ln(1+a_{k})\
& geq & sum_{k=1}^{n}ln(1+b_{k})\
& geq & frac{1}{2}sum_{k=1}^{n}b_{k}.
end{eqnarray*}

If $sum_{k=1}^{infty}a_{k}=infty$, we will have $sum_{k=1}^{infty}b_{k}=infty$.
(For, consider two cases. Case 1: There exists $N$ such that $a_{k}leq 1$
whenever $kgeq N$. In this case, $sum_{k=1}^{infty}b_{k}geqsum_{k=N}^{infty}b_{k}=sum_{k=N}^{infty}a_{k}=infty$.
Case 2: There does not exist $N$ such that $a_{k}leq 1$ whenever $kgeq N$.
Then, there exists a subsequence $(a_{k_{l}})_{l}$ such that $a_{k_{l}}>1$.
We have that $sum_{k=1}^{infty}b_{k}geqsum_{l=1}^{infty}b_{k_{l}}=sum_{l=1}^{infty}1=infty$.)
Therefore, $-ln P_{n}rightarrowinfty$ and hence $exp(-ln P_{n})rightarrowinfty$.
But $exp(-ln P_{n})=frac{1}{P_{n}}$. It follows that $P_{n}rightarrow0$.






share|cite|improve this answer











$endgroup$














    Your Answer








    StackExchange.ready(function() {
    var channelOptions = {
    tags: "".split(" "),
    id: "69"
    };
    initTagRenderer("".split(" "), "".split(" "), channelOptions);

    StackExchange.using("externalEditor", function() {
    // Have to fire editor after snippets, if snippets enabled
    if (StackExchange.settings.snippets.snippetsEnabled) {
    StackExchange.using("snippets", function() {
    createEditor();
    });
    }
    else {
    createEditor();
    }
    });

    function createEditor() {
    StackExchange.prepareEditor({
    heartbeatType: 'answer',
    autoActivateHeartbeat: false,
    convertImagesToLinks: true,
    noModals: true,
    showLowRepImageUploadWarning: true,
    reputationToPostImages: 10,
    bindNavPrevention: true,
    postfix: "",
    imageUploader: {
    brandingHtml: "Powered by u003ca class="icon-imgur-white" href="https://imgur.com/"u003eu003c/au003e",
    contentPolicyHtml: "User contributions licensed under u003ca href="https://creativecommons.org/licenses/by-sa/3.0/"u003ecc by-sa 3.0 with attribution requiredu003c/au003e u003ca href="https://stackoverflow.com/legal/content-policy"u003e(content policy)u003c/au003e",
    allowUrls: true
    },
    noCode: true, onDemand: true,
    discardSelector: ".discard-answer"
    ,immediatelyShowMarkdownHelp:true
    });


    }
    });














    draft saved

    draft discarded


















    StackExchange.ready(
    function () {
    StackExchange.openid.initPostLogin('.new-post-login', 'https%3a%2f%2fmath.stackexchange.com%2fquestions%2f3058029%2ffor-a-n-positive-prod-n-frac11a-n-0-if-and-only-if-sum-n-a-n-infty%23new-answer', 'question_page');
    }
    );

    Post as a guest















    Required, but never shown

























    3 Answers
    3






    active

    oldest

    votes








    3 Answers
    3






    active

    oldest

    votes









    active

    oldest

    votes






    active

    oldest

    votes









    4












    $begingroup$

    I assume $lim_na_n=0$ otherwise it is trivial: then
    $$
    lnPi_nbig({1over{1+ a_n}}big)=sum -ln(1+a_n),
    $$

    and this is equivalent to $-sum a_n$ since
    $$ln(1+a_n)simeq a_ntext{ if }lim_na_n=0.$$






    share|cite|improve this answer











    $endgroup$













    • $begingroup$
      So $-sum_nlog(1+a_n)=-infty$ would imply that the product is zero, and being finite would imply that the product is positive? Thanks.
      $endgroup$
      – Math1000
      Dec 31 '18 at 20:55
















    4












    $begingroup$

    I assume $lim_na_n=0$ otherwise it is trivial: then
    $$
    lnPi_nbig({1over{1+ a_n}}big)=sum -ln(1+a_n),
    $$

    and this is equivalent to $-sum a_n$ since
    $$ln(1+a_n)simeq a_ntext{ if }lim_na_n=0.$$






    share|cite|improve this answer











    $endgroup$













    • $begingroup$
      So $-sum_nlog(1+a_n)=-infty$ would imply that the product is zero, and being finite would imply that the product is positive? Thanks.
      $endgroup$
      – Math1000
      Dec 31 '18 at 20:55














    4












    4








    4





    $begingroup$

    I assume $lim_na_n=0$ otherwise it is trivial: then
    $$
    lnPi_nbig({1over{1+ a_n}}big)=sum -ln(1+a_n),
    $$

    and this is equivalent to $-sum a_n$ since
    $$ln(1+a_n)simeq a_ntext{ if }lim_na_n=0.$$






    share|cite|improve this answer











    $endgroup$



    I assume $lim_na_n=0$ otherwise it is trivial: then
    $$
    lnPi_nbig({1over{1+ a_n}}big)=sum -ln(1+a_n),
    $$

    and this is equivalent to $-sum a_n$ since
    $$ln(1+a_n)simeq a_ntext{ if }lim_na_n=0.$$







    share|cite|improve this answer














    share|cite|improve this answer



    share|cite|improve this answer








    edited Jan 1 at 0:00









    Daniele Tampieri

    2,75721022




    2,75721022










    answered Dec 31 '18 at 20:43









    Tsemo AristideTsemo Aristide

    60.9k11446




    60.9k11446












    • $begingroup$
      So $-sum_nlog(1+a_n)=-infty$ would imply that the product is zero, and being finite would imply that the product is positive? Thanks.
      $endgroup$
      – Math1000
      Dec 31 '18 at 20:55


















    • $begingroup$
      So $-sum_nlog(1+a_n)=-infty$ would imply that the product is zero, and being finite would imply that the product is positive? Thanks.
      $endgroup$
      – Math1000
      Dec 31 '18 at 20:55
















    $begingroup$
    So $-sum_nlog(1+a_n)=-infty$ would imply that the product is zero, and being finite would imply that the product is positive? Thanks.
    $endgroup$
    – Math1000
    Dec 31 '18 at 20:55




    $begingroup$
    So $-sum_nlog(1+a_n)=-infty$ would imply that the product is zero, and being finite would imply that the product is positive? Thanks.
    $endgroup$
    – Math1000
    Dec 31 '18 at 20:55











    1












    $begingroup$

    Let $a_ngeq 0$ for all $n.$ By induction on $n$ we have $$prod_{j=1}(1+a_j)geq 1+sum_{j=1}^na_j.$$ So if the sum diverges then so does the product.



    We have $exp(a_j)geq 1+a_j, $ so $$exp (sum_{j=1}^na_j)geq prod_{j=1}^n(1+a_j).$$ So if the sum converges then so does the product.






    share|cite|improve this answer









    $endgroup$













    • $begingroup$
      I think you're looking at $prod_j (1+a_j)$ as opposed to $prod_j frac1{1+a_j}$ here.
      $endgroup$
      – Math1000
      Dec 31 '18 at 22:05
















    1












    $begingroup$

    Let $a_ngeq 0$ for all $n.$ By induction on $n$ we have $$prod_{j=1}(1+a_j)geq 1+sum_{j=1}^na_j.$$ So if the sum diverges then so does the product.



    We have $exp(a_j)geq 1+a_j, $ so $$exp (sum_{j=1}^na_j)geq prod_{j=1}^n(1+a_j).$$ So if the sum converges then so does the product.






    share|cite|improve this answer









    $endgroup$













    • $begingroup$
      I think you're looking at $prod_j (1+a_j)$ as opposed to $prod_j frac1{1+a_j}$ here.
      $endgroup$
      – Math1000
      Dec 31 '18 at 22:05














    1












    1








    1





    $begingroup$

    Let $a_ngeq 0$ for all $n.$ By induction on $n$ we have $$prod_{j=1}(1+a_j)geq 1+sum_{j=1}^na_j.$$ So if the sum diverges then so does the product.



    We have $exp(a_j)geq 1+a_j, $ so $$exp (sum_{j=1}^na_j)geq prod_{j=1}^n(1+a_j).$$ So if the sum converges then so does the product.






    share|cite|improve this answer









    $endgroup$



    Let $a_ngeq 0$ for all $n.$ By induction on $n$ we have $$prod_{j=1}(1+a_j)geq 1+sum_{j=1}^na_j.$$ So if the sum diverges then so does the product.



    We have $exp(a_j)geq 1+a_j, $ so $$exp (sum_{j=1}^na_j)geq prod_{j=1}^n(1+a_j).$$ So if the sum converges then so does the product.







    share|cite|improve this answer












    share|cite|improve this answer



    share|cite|improve this answer










    answered Dec 31 '18 at 21:11









    DanielWainfleetDanielWainfleet

    36k31648




    36k31648












    • $begingroup$
      I think you're looking at $prod_j (1+a_j)$ as opposed to $prod_j frac1{1+a_j}$ here.
      $endgroup$
      – Math1000
      Dec 31 '18 at 22:05


















    • $begingroup$
      I think you're looking at $prod_j (1+a_j)$ as opposed to $prod_j frac1{1+a_j}$ here.
      $endgroup$
      – Math1000
      Dec 31 '18 at 22:05
















    $begingroup$
    I think you're looking at $prod_j (1+a_j)$ as opposed to $prod_j frac1{1+a_j}$ here.
    $endgroup$
    – Math1000
    Dec 31 '18 at 22:05




    $begingroup$
    I think you're looking at $prod_j (1+a_j)$ as opposed to $prod_j frac1{1+a_j}$ here.
    $endgroup$
    – Math1000
    Dec 31 '18 at 22:05











    0












    $begingroup$

    Fact 1: For any $xgeq0$, we have $ln(1+x)leq x$.



    Proof: Let $f:[0,infty)rightarrowmathbb{R}$ be defined by $f(x)=x-ln(1+x)$.
    Observe that $f'(x)=frac{x}{1+x}geq0,$ so $f$ is increasing. For
    any $xin[0,infty)$, we have $f(x)geq f(0)=0$.



    /////////////////////////////////



    Fact 2: For any $xin[0,1]$, we have $ln(1+x)geqfrac{x}{2}$.



    Proof: Let $g:[0,1]rightarrowmathbb{R}$ be defined by $g(x)=ln(1+x)-frac{x}{2}$.
    Observe that $g'(x)=frac{1-x}{2(1+x)}geq0$, so $g$ is increasing.
    For any $xin[0,1]$, we have $g(x)geq g(0)=0$.



    /////////////////////////////////



    Let $P_{n}=prod_{k=1}^{n}frac{1}{1+a_{k}}$. Then
    begin{eqnarray*}
    -ln P_{n} & = & sum_{k=1}^{n}ln(1+a_{k})\
    & leq & sum_{k=1}^{n}a_{k}.
    end{eqnarray*}

    If $P_{n}rightarrow0$, we have $-ln P_{n}rightarrowinfty$, so
    $sum_{k=1}^{infty}a_{k}=infty$.



    /////////////////////////////////////////////////



    For each $k$, let $b_{k}=min(a_{k},1)leq a_{k}$. Then,
    begin{eqnarray*}
    -ln P_{n} & = & sum_{k=1}^{n}ln(1+a_{k})\
    & geq & sum_{k=1}^{n}ln(1+b_{k})\
    & geq & frac{1}{2}sum_{k=1}^{n}b_{k}.
    end{eqnarray*}

    If $sum_{k=1}^{infty}a_{k}=infty$, we will have $sum_{k=1}^{infty}b_{k}=infty$.
    (For, consider two cases. Case 1: There exists $N$ such that $a_{k}leq 1$
    whenever $kgeq N$. In this case, $sum_{k=1}^{infty}b_{k}geqsum_{k=N}^{infty}b_{k}=sum_{k=N}^{infty}a_{k}=infty$.
    Case 2: There does not exist $N$ such that $a_{k}leq 1$ whenever $kgeq N$.
    Then, there exists a subsequence $(a_{k_{l}})_{l}$ such that $a_{k_{l}}>1$.
    We have that $sum_{k=1}^{infty}b_{k}geqsum_{l=1}^{infty}b_{k_{l}}=sum_{l=1}^{infty}1=infty$.)
    Therefore, $-ln P_{n}rightarrowinfty$ and hence $exp(-ln P_{n})rightarrowinfty$.
    But $exp(-ln P_{n})=frac{1}{P_{n}}$. It follows that $P_{n}rightarrow0$.






    share|cite|improve this answer











    $endgroup$


















      0












      $begingroup$

      Fact 1: For any $xgeq0$, we have $ln(1+x)leq x$.



      Proof: Let $f:[0,infty)rightarrowmathbb{R}$ be defined by $f(x)=x-ln(1+x)$.
      Observe that $f'(x)=frac{x}{1+x}geq0,$ so $f$ is increasing. For
      any $xin[0,infty)$, we have $f(x)geq f(0)=0$.



      /////////////////////////////////



      Fact 2: For any $xin[0,1]$, we have $ln(1+x)geqfrac{x}{2}$.



      Proof: Let $g:[0,1]rightarrowmathbb{R}$ be defined by $g(x)=ln(1+x)-frac{x}{2}$.
      Observe that $g'(x)=frac{1-x}{2(1+x)}geq0$, so $g$ is increasing.
      For any $xin[0,1]$, we have $g(x)geq g(0)=0$.



      /////////////////////////////////



      Let $P_{n}=prod_{k=1}^{n}frac{1}{1+a_{k}}$. Then
      begin{eqnarray*}
      -ln P_{n} & = & sum_{k=1}^{n}ln(1+a_{k})\
      & leq & sum_{k=1}^{n}a_{k}.
      end{eqnarray*}

      If $P_{n}rightarrow0$, we have $-ln P_{n}rightarrowinfty$, so
      $sum_{k=1}^{infty}a_{k}=infty$.



      /////////////////////////////////////////////////



      For each $k$, let $b_{k}=min(a_{k},1)leq a_{k}$. Then,
      begin{eqnarray*}
      -ln P_{n} & = & sum_{k=1}^{n}ln(1+a_{k})\
      & geq & sum_{k=1}^{n}ln(1+b_{k})\
      & geq & frac{1}{2}sum_{k=1}^{n}b_{k}.
      end{eqnarray*}

      If $sum_{k=1}^{infty}a_{k}=infty$, we will have $sum_{k=1}^{infty}b_{k}=infty$.
      (For, consider two cases. Case 1: There exists $N$ such that $a_{k}leq 1$
      whenever $kgeq N$. In this case, $sum_{k=1}^{infty}b_{k}geqsum_{k=N}^{infty}b_{k}=sum_{k=N}^{infty}a_{k}=infty$.
      Case 2: There does not exist $N$ such that $a_{k}leq 1$ whenever $kgeq N$.
      Then, there exists a subsequence $(a_{k_{l}})_{l}$ such that $a_{k_{l}}>1$.
      We have that $sum_{k=1}^{infty}b_{k}geqsum_{l=1}^{infty}b_{k_{l}}=sum_{l=1}^{infty}1=infty$.)
      Therefore, $-ln P_{n}rightarrowinfty$ and hence $exp(-ln P_{n})rightarrowinfty$.
      But $exp(-ln P_{n})=frac{1}{P_{n}}$. It follows that $P_{n}rightarrow0$.






      share|cite|improve this answer











      $endgroup$
















        0












        0








        0





        $begingroup$

        Fact 1: For any $xgeq0$, we have $ln(1+x)leq x$.



        Proof: Let $f:[0,infty)rightarrowmathbb{R}$ be defined by $f(x)=x-ln(1+x)$.
        Observe that $f'(x)=frac{x}{1+x}geq0,$ so $f$ is increasing. For
        any $xin[0,infty)$, we have $f(x)geq f(0)=0$.



        /////////////////////////////////



        Fact 2: For any $xin[0,1]$, we have $ln(1+x)geqfrac{x}{2}$.



        Proof: Let $g:[0,1]rightarrowmathbb{R}$ be defined by $g(x)=ln(1+x)-frac{x}{2}$.
        Observe that $g'(x)=frac{1-x}{2(1+x)}geq0$, so $g$ is increasing.
        For any $xin[0,1]$, we have $g(x)geq g(0)=0$.



        /////////////////////////////////



        Let $P_{n}=prod_{k=1}^{n}frac{1}{1+a_{k}}$. Then
        begin{eqnarray*}
        -ln P_{n} & = & sum_{k=1}^{n}ln(1+a_{k})\
        & leq & sum_{k=1}^{n}a_{k}.
        end{eqnarray*}

        If $P_{n}rightarrow0$, we have $-ln P_{n}rightarrowinfty$, so
        $sum_{k=1}^{infty}a_{k}=infty$.



        /////////////////////////////////////////////////



        For each $k$, let $b_{k}=min(a_{k},1)leq a_{k}$. Then,
        begin{eqnarray*}
        -ln P_{n} & = & sum_{k=1}^{n}ln(1+a_{k})\
        & geq & sum_{k=1}^{n}ln(1+b_{k})\
        & geq & frac{1}{2}sum_{k=1}^{n}b_{k}.
        end{eqnarray*}

        If $sum_{k=1}^{infty}a_{k}=infty$, we will have $sum_{k=1}^{infty}b_{k}=infty$.
        (For, consider two cases. Case 1: There exists $N$ such that $a_{k}leq 1$
        whenever $kgeq N$. In this case, $sum_{k=1}^{infty}b_{k}geqsum_{k=N}^{infty}b_{k}=sum_{k=N}^{infty}a_{k}=infty$.
        Case 2: There does not exist $N$ such that $a_{k}leq 1$ whenever $kgeq N$.
        Then, there exists a subsequence $(a_{k_{l}})_{l}$ such that $a_{k_{l}}>1$.
        We have that $sum_{k=1}^{infty}b_{k}geqsum_{l=1}^{infty}b_{k_{l}}=sum_{l=1}^{infty}1=infty$.)
        Therefore, $-ln P_{n}rightarrowinfty$ and hence $exp(-ln P_{n})rightarrowinfty$.
        But $exp(-ln P_{n})=frac{1}{P_{n}}$. It follows that $P_{n}rightarrow0$.






        share|cite|improve this answer











        $endgroup$



        Fact 1: For any $xgeq0$, we have $ln(1+x)leq x$.



        Proof: Let $f:[0,infty)rightarrowmathbb{R}$ be defined by $f(x)=x-ln(1+x)$.
        Observe that $f'(x)=frac{x}{1+x}geq0,$ so $f$ is increasing. For
        any $xin[0,infty)$, we have $f(x)geq f(0)=0$.



        /////////////////////////////////



        Fact 2: For any $xin[0,1]$, we have $ln(1+x)geqfrac{x}{2}$.



        Proof: Let $g:[0,1]rightarrowmathbb{R}$ be defined by $g(x)=ln(1+x)-frac{x}{2}$.
        Observe that $g'(x)=frac{1-x}{2(1+x)}geq0$, so $g$ is increasing.
        For any $xin[0,1]$, we have $g(x)geq g(0)=0$.



        /////////////////////////////////



        Let $P_{n}=prod_{k=1}^{n}frac{1}{1+a_{k}}$. Then
        begin{eqnarray*}
        -ln P_{n} & = & sum_{k=1}^{n}ln(1+a_{k})\
        & leq & sum_{k=1}^{n}a_{k}.
        end{eqnarray*}

        If $P_{n}rightarrow0$, we have $-ln P_{n}rightarrowinfty$, so
        $sum_{k=1}^{infty}a_{k}=infty$.



        /////////////////////////////////////////////////



        For each $k$, let $b_{k}=min(a_{k},1)leq a_{k}$. Then,
        begin{eqnarray*}
        -ln P_{n} & = & sum_{k=1}^{n}ln(1+a_{k})\
        & geq & sum_{k=1}^{n}ln(1+b_{k})\
        & geq & frac{1}{2}sum_{k=1}^{n}b_{k}.
        end{eqnarray*}

        If $sum_{k=1}^{infty}a_{k}=infty$, we will have $sum_{k=1}^{infty}b_{k}=infty$.
        (For, consider two cases. Case 1: There exists $N$ such that $a_{k}leq 1$
        whenever $kgeq N$. In this case, $sum_{k=1}^{infty}b_{k}geqsum_{k=N}^{infty}b_{k}=sum_{k=N}^{infty}a_{k}=infty$.
        Case 2: There does not exist $N$ such that $a_{k}leq 1$ whenever $kgeq N$.
        Then, there exists a subsequence $(a_{k_{l}})_{l}$ such that $a_{k_{l}}>1$.
        We have that $sum_{k=1}^{infty}b_{k}geqsum_{l=1}^{infty}b_{k_{l}}=sum_{l=1}^{infty}1=infty$.)
        Therefore, $-ln P_{n}rightarrowinfty$ and hence $exp(-ln P_{n})rightarrowinfty$.
        But $exp(-ln P_{n})=frac{1}{P_{n}}$. It follows that $P_{n}rightarrow0$.







        share|cite|improve this answer














        share|cite|improve this answer



        share|cite|improve this answer








        edited Dec 31 '18 at 21:16

























        answered Dec 31 '18 at 21:10









        Danny Pak-Keung ChanDanny Pak-Keung Chan

        2,56938




        2,56938






























            draft saved

            draft discarded




















































            Thanks for contributing an answer to Mathematics Stack Exchange!


            • Please be sure to answer the question. Provide details and share your research!

            But avoid



            • Asking for help, clarification, or responding to other answers.

            • Making statements based on opinion; back them up with references or personal experience.


            Use MathJax to format equations. MathJax reference.


            To learn more, see our tips on writing great answers.




            draft saved


            draft discarded














            StackExchange.ready(
            function () {
            StackExchange.openid.initPostLogin('.new-post-login', 'https%3a%2f%2fmath.stackexchange.com%2fquestions%2f3058029%2ffor-a-n-positive-prod-n-frac11a-n-0-if-and-only-if-sum-n-a-n-infty%23new-answer', 'question_page');
            }
            );

            Post as a guest















            Required, but never shown





















































            Required, but never shown














            Required, but never shown












            Required, but never shown







            Required, but never shown

































            Required, but never shown














            Required, but never shown












            Required, but never shown







            Required, but never shown







            Popular posts from this blog

            Biblatex bibliography style without URLs when DOI exists (in Overleaf with Zotero bibliography)

            ComboBox Display Member on multiple fields

            Is it possible to collect Nectar points via Trainline?